[MIX] Zestaw mola (końcówka)

Zadania z kółek matematycznych lub obozów przygotowujących do OM. Problemy z minionych olimpiad i konkursów matematycznych.
Regulamin forum
Wszystkie tematy znajdujące się w tym dziale powinny być tagowane tj. posiadać przedrostek postaci [Nierówności], [Planimetria], itp.. Temat może posiadać wiele różnych tagów. Nazwa tematu nie może składać się z samych tagów.
Awatar użytkownika
mol_ksiazkowy
Użytkownik
Użytkownik
Posty: 11415
Rejestracja: 9 maja 2006, o 12:35
Płeć: Mężczyzna
Lokalizacja: Kraków
Podziękował: 3155 razy
Pomógł: 748 razy

[MIX] Zestaw mola (końcówka)

Post autor: mol_ksiazkowy »

1. Niech \(\displaystyle{ m}\) i \(\displaystyle{ n}\) bedą liczbami naturalnymi. Wykazać iż \(\displaystyle{ (2^m - 1)^2}\) dzieli \(\displaystyle{ 2^n -1}\) wtedy i tylko wtedy, gdy \(\displaystyle{ m(2^m - 1)}\) dzieli \(\displaystyle{ n}\)
2. Niech \(\displaystyle{ a}\) będzie liczbą całkowitą dodatnią taka, iż dla każdego \(\displaystyle{ n \in N}\): \(\displaystyle{ a + n^2}\) jest sumą kwadratów liczb całkowitych \(\displaystyle{ x}\) i \(\displaystyle{ y}\). Udowodnić, iż \(\displaystyle{ a}\) jest kwadratem liczby całkowitej.
3. Dane są liczby naturalne \(\displaystyle{ n> 4}\) i \(\displaystyle{ k}\). Dowieść że jeżeli \(\displaystyle{ n > k^2}\) i \(\displaystyle{ n}\) nie jest liczbą pierwsza, to \(\displaystyle{ n}\) dzieli \(\displaystyle{ (n - k - 1)!}\).
Wskazać iż założenie: „\(\displaystyle{ n}\) nie jest liczbą pierwszą” jest tu istotne
4. Niech \(\displaystyle{ p}\) będzie liczba pierwszą \(\displaystyle{ p \equiv 2 \ (mod \ 3 )}\) oraz p dzieli \(\displaystyle{ a^2 + ab + b^2}\) gdzie \(\displaystyle{ a}\) i \(\displaystyle{ b}\) są to liczby całkowite. Udowodnić, że obie \(\displaystyle{ a}\) i \(\displaystyle{ b}\) są podzielne przez \(\displaystyle{ p}\)
104 ntp
5. Wykazać, że jeśli \(\displaystyle{ a, b}\) są liczbami całkowitymi zaś \(\displaystyle{ n}\) jest liczbą naturalną, to \(\displaystyle{ a^n \equiv b^n \ (mod \ n^2)}\). Czy twierdzenie to można odwrócić ?
104 ntp
6. Ile jest równa suma wszystkich liczb \(\displaystyle{ \frac{a}{b}}\) gdzie \(\displaystyle{ a}\) oraz \(\displaystyle{ b}\) są względnie pierwsze i są one dodatnimi dzielnikami \(\displaystyle{ 27 000}\) ?
104 ntp

7. Niech \(\displaystyle{ p}\) będzie liczba pierwsza taka, iż \(\displaystyle{ p \equiv 5 \ (mod \ 8)}\). Dowieść, że gdy \(\displaystyle{ p}\) dzieli \(\displaystyle{ x^4 + y^4}\) dla \(\displaystyle{ x, y}\) bedacych liczbami całkowitymi, to \(\displaystyle{ p}\) dzieli \(\displaystyle{ xy}\)
8. Liczby \(\displaystyle{ a}\) i \(\displaystyle{ b}\) są naturalne oraz \(\displaystyle{ \sqrt{3} < \frac{a}{b} < \frac{26}{15}}\). Udowodnić, iż \(\displaystyle{ b > 41}\)
9. Wykazać, że nie istnieją liczby całkowite \(\displaystyle{ x, y, z}\) takie, że \(\displaystyle{ 13= x^3 +y^3 +z^3}\)
wn
10. Wykazać, że dla dowolnej liczby naturalnej \(\displaystyle{ m}\) istnieje nieskończenie wiele niepodzielnych przez \(\displaystyle{ 10}\) liczb naturalnych \(\displaystyle{ n}\) takich, że suma cyfr liczby \(\displaystyle{ n}\) równa jest sumie cyfr liczby \(\displaystyle{ mn}\)
11. Niech \(\displaystyle{ p>2}\) będzie liczbą pierwszą, i \(\displaystyle{ a^{-1}=x}\) gdy \(\displaystyle{ ax \equiv 1 \ (mod \ p)}\). Wykazać, iż \(\displaystyle{ \frac{2^{p-1}- 1}{p} \equiv 1^{-1} + 3^{-1}+ ... + (p-2)^{-1} \ (mod \ p)}\)
12. a) Udowodnić, że liczby naturalne \(\displaystyle{ 8k + 3}\) oraz \(\displaystyle{ 8k + 5}\) nie są równe \(\displaystyle{ x^2 - 2y^2}\), gdzie \(\displaystyle{ x, y}\) są to liczby całkowite
b) Rozstrzygnąć, czy jeśli \(\displaystyle{ n}\) daje przy dzieleniu przez \(\displaystyle{ 8}\) resztę różną od \(\displaystyle{ 3}\) i od \(\displaystyle{ 5}\) to może być \(\displaystyle{ n = x^2 - 2y^2}\) gdzie \(\displaystyle{ x, y}\) jw.
ws
13. Niech \(\displaystyle{ p}\) będzie liczbą pierwszą, zaś \(\displaystyle{ a, b, c}\) są liczbami naturalnymi mniejszymi od \(\displaystyle{ p}\) i takimi, że \(\displaystyle{ a^3 \equiv b^3 \equiv c^3 \ (mod \ p)}\). Dowieść, iż \(\displaystyle{ a+b+c}\) dzieli \(\displaystyle{ a^2 + b^2 +c^2}\)
14. a) Wykazać, iż jeśli liczby \(\displaystyle{ a}\) i \(\displaystyle{ m}\) są względnie pierwsze to \(\displaystyle{ x = ba^{\phi(m) - 1}}\) jest rozwiązaniem kongruencji \(\displaystyle{ ax \equiv b \ (mod \ m)}\)
b) rozwiązać \(\displaystyle{ 19x \equiv 1 \ (mod \ 140)}\)
m-z
15. Niech \(\displaystyle{ n}\) będzie liczba naturalną nieparzystą, \(\displaystyle{ n>3}\) oraz \(\displaystyle{ \begin{cases} k = min \ x : nx+1 \in K \\t=min x \in N : nx \in K \end{cases}}\)
gdzie \(\displaystyle{ K = \{ 1^2, 2^2, 3^2, .... \}}\)
Wykazać, że \(\displaystyle{ n}\) jest liczbą pierwszą tylko wtedy (warunek równoważny) gdy \(\displaystyle{ 4k > n}\) oraz \(\displaystyle{ 4t > n}\)
m-z
16. Czy dla każdej liczby naturalnej \(\displaystyle{ m}\) istnieją liczby \(\displaystyle{ k, l}\); \(\displaystyle{ k \neq l}\), o tej własności iż \(\displaystyle{ k^m}\) i \(\displaystyle{ l^m}\) różnią się jedynie kolejnością cyfr ? ; np. gdy \(\displaystyle{ m=2}\) to \(\displaystyle{ k=12 \ l=21}\) etc.
m
17. Mając dane \(\displaystyle{ n= 188 \ 686 \ 013 = pq}\) gdzie \(\displaystyle{ p}\) i \(\displaystyle{ q}\) są liczbami pierwszymi takimi, iż \(\displaystyle{ 100|\frac{p}{q} - 3| < 1}\)
znaleźć \(\displaystyle{ p}\) i \(\displaystyle{ q}\)

18. Niech \(\displaystyle{ x, y, n, k, p}\) będą liczbami naturalnymi oraz \(\displaystyle{ x^n + y^n = p^k}\) i \(\displaystyle{ n>1}\) jest liczbą nieparzystą, zaś \(\displaystyle{ p>2}\) to liczba pierwsza. Udowodnić, że \(\displaystyle{ n}\) jest potęgą naturalną \(\displaystyle{ p}\)
19. Wykazać, iż jeśli dane są trzy liczby nieparzyste, to istnieje też taka liczba całkowita, iż suma kwadratów tych czterech liczb jest kwadratem liczby całkowitej
ex
20. Udowodnić, iż nie istnieje czwórka \(\displaystyle{ (x, y, z, w)}\) parami różnych liczb naturalnych, i takich iż liczby: \(\displaystyle{ x^2 + y + z + w, \ x + y^2 + z +w , \ x + y+ z^2 + w , \ x + y + z + w^2}\)
są kwadratami liczb całkowitych; jednakże istnieją inne czwórki np. \(\displaystyle{ (96, 96, 57, 40)}\). Wyznaczyć je wszystkie
21. Udowodnić, iż każda liczba całkowita \(\displaystyle{ n \geq 7}\) jest suma dwóch względnie pierwszych liczb naturalnych większych od \(\displaystyle{ 1}\)
22. Wyznaczyć wszystkie trójki \(\displaystyle{ (p, q, r)}\) liczb całkowitych, takich że \(\displaystyle{ \frac{1}{p - q - r} = \frac{1}{q} + \frac{1}{r}}\) oraz liczby \(\displaystyle{ |p|, |q|, |r|}\) są pierwsze

23. Dowieść, że jeśli \(\displaystyle{ n}\) jest liczba naturalna to istnieją liczby całkowite \(\displaystyle{ x, y, z}\) takie że \(\displaystyle{ n = x^2 + y^2 - 5z^2}\)
wn
24. Wyznaczyć wszystkie liczby naturalne \(\displaystyle{ m}\) takie, że \(\displaystyle{ m^5 + m+ 1}\) oraz \(\displaystyle{ m^{11} + m +1}\) są liczbami pierwszymi.

25. Niech \(\displaystyle{ n}\) i \(\displaystyle{ p}\) będą liczbami naturalnymi ; zaś \(\displaystyle{ p}\) jest pierwsza; takimi że liczby \(\displaystyle{ 2np - 1}\) i \(\displaystyle{ 2np + 1}\) są pierwsze. Udowodnić, że \(\displaystyle{ p=3}\)

26. Wykazać, iż dla dowolnej liczby naturalnej \(\displaystyle{ n}\) istnieje liczba naturalna \(\displaystyle{ m}\) taka, że \(\displaystyle{ n}\) dzieli \(\displaystyle{ 2^m - m}\); oraz że jeśli \(\displaystyle{ n}\) jest liczbą pierwszą, to takich \(\displaystyle{ m}\) jest nieskończenie wiele
27. Udowodnić, iż w ciągu: \(\displaystyle{ 1, 31, 331, 3331, ....}\) jest nieskończenie wiele liczb złożonych i znaleźć najmniejszą z nich.
ws

28. Znaleźć największa liczbę naturalną \(\displaystyle{ n}\) o tej własności, iż w każdym zbiorze mającym \(\displaystyle{ 12}\) elementów, będących kolejnymi liczbami naturalnymi istnieje liczba niepodzielna przez żadną z początkowych liczb pierwszych \(\displaystyle{ p_1, p_2, ..., p_n}\)

29. Czy istnieje nieskończenie wiele liczb \(\displaystyle{ a}\), \(\displaystyle{ b}\) oraz \(\displaystyle{ a \neq b}\) takich że \(\displaystyle{ \frac{a^2 +b}{b^2 +a}}\) jest liczbą całkowitą ?

30. Niech \(\displaystyle{ n >2}\) będzie liczbą całkowitą, zaś \(\displaystyle{ n^2=(m+1)^3 - m^3}\) gdzie \(\displaystyle{ m}\) jest liczbą naturalną. Udowodnić że \(\displaystyle{ n}\) jest sumą kwadratów dwóch liczb całkowitych.

31. Udowodnić, że gdy \(\displaystyle{ a_1 < a_2 < ... < a_{n - 1} < a_n}\) to \(\displaystyle{ NWW(a_1, a_2, ..., a_{n - 1}, a_n ) \geq na_1}\)
32. Wykazać, że istnieje nieskończenie wiele par liczb naturalnych \(\displaystyle{ m}\) i \(\displaystyle{ n}\); \(\displaystyle{ m \neq n}\) takich że:
\(\displaystyle{ \phi(m + n) = \phi(m) + \phi(n)}\);
gdzie \(\displaystyle{ \phi()}\) jest funkcją Eulera

33. Niech \(\displaystyle{ G_n = 2^{3^n} + 1}\) dla \(\displaystyle{ n \geq 1}\)
Wykazać, że liczby \(\displaystyle{ G_n}\) są złożone, oraz że:
a) \(\displaystyle{ 3^{n+1}}\) dzieli \(\displaystyle{ G_n}\)
b) jeśli \(\displaystyle{ p}\) jest liczbą pierwszą, która dzieli \(\displaystyle{ G_n}\) to \(\displaystyle{ p}\) dzieli też \(\displaystyle{ G_{n+1}}\)
34. a) Mając dane \(\displaystyle{ 34! = 295 \ 232 \ 799 \ cd9 \ 604 \ 140 \ 847 \ 618 \ 609 \ 643 \ 5ab \ 000 \ 000}\)
Wyznaczyć \(\displaystyle{ a, b, c, d}\)
35. Wyznaczyć najmniejszą liczbę naturalną \(\displaystyle{ n}\) o tej własności, iż liczby \(\displaystyle{ \sqrt{\frac{n}{2}}, \ \sqrt[7]{\frac{n}{7}}, \ \sqrt[11]{\frac{n}{11}}}\) są całkowite
36. Znaleźć wszystkie możliwe trójki \(\displaystyle{ (a,b,c)}\) liczb całkowitych, spełniających: \(\displaystyle{ 1<a < b <c}\) i takich iż \(\displaystyle{ (a-1)(b-1)(c-1)}\) dzieli \(\displaystyle{ abc - 1}\)
imo92
37. Dowieść, że gdy \(\displaystyle{ n}\) jest liczbą naturalną, to \(\displaystyle{ \lfloor (\frac{1+ \sqrt{5}}{2})^{4n-2} \rfloor}\) jest kwadratem liczby całkowitej

38. Jeśli \(\displaystyle{ a, b}\) są liczbami naturalnymi oraz \(\displaystyle{ p=\frac{b}{4}\sqrt{\frac{2a-b}{2a+b}}}\) jest liczbą pierwszą. Jaka jest największa możliwa wartość \(\displaystyle{ p}\) ?

39. Dla jakich \(\displaystyle{ n}\) w ciągu \(\displaystyle{ n, \tau(n), \tau(\tau(n)), ....}\)
nie ma kwadratu liczby całkowitej ?
gdzie \(\displaystyle{ \tau(n)}\) jest to ilość dzielników (dodatnich) liczby \(\displaystyle{ n}\)
40. Czy z tożsamości Ramanujana:
\(\displaystyle{ (3a^2 + 5ab - 5b^2)^3 + (4a^2 - 4ab - 5b^2)^3 = (- 5a^2 + 5ab + 3b^2)^3 + (6a^2 - 4ab + 4b^2 )^3}\)
można uzyskać rozkład \(\displaystyle{ 1729 = 1^3 + 12^3 = 9^3 + 10^3}\) (liczba Hardy’ego ) znajdując w niej \(\displaystyle{ a}\) i \(\displaystyle{ b}\) ?

41. 7 x dzielniki; Wyznaczyć:
a) liczbę całkowitą \(\displaystyle{ k}\) taką, że \(\displaystyle{ 10^{k}= 2*3^2*7*47*168861871 - 62}\)
b) trzy dzielniki pierwsze \(\displaystyle{ 10^{27} + 1}\)
c) najmniejszy dzielnik pierwszy \(\displaystyle{ 2^{2^9} + 1}\)
d) rozkład na czynniki \(\displaystyle{ 2^{71} - 1}\)
e) największy dzielnik pierwszy \(\displaystyle{ 2^{2^6}+ 1}\)
f*) wykazać, że \(\displaystyle{ 2^{1992} - 1}\) jest iloczynem sześciu liczb naturalnych większych od \(\displaystyle{ 2^{248}}\)
g) rozkład na czynniki \(\displaystyle{ n=48 \ 790 \ 373}\) mając dane \(\displaystyle{ 7n = 699^3 + 8^3}\)

42. Trzy liczby \(\displaystyle{ a, b, c}\)rzeczywiste ale takie, iż \(\displaystyle{ a - b, \ a^2 - b^2, \ a^3 - b^3}\) są całkowite. Czy z tego wynika iż \(\displaystyle{ a}\) oraz \(\displaystyle{ b}\) są całkowite ?

43. Niech \(\displaystyle{ a, b}\) oraz \(\displaystyle{ \frac{b+1}{a} , \ \frac{a^2 - 2}{b}}\) będą liczbami całkowitymi Udowodnić, iż \(\displaystyle{ \frac{b+1}{2}}\) jest kwadratem liczby całkowitej
44. Liczby \(\displaystyle{ x, y, z}\) są całkowite oraz \(\displaystyle{ \frac{xy}{z} + \frac{xz}{y} +\frac{yz}{x} =3}\). Czy z tego wynika, że \(\displaystyle{ x=y=z=1}\) ?

45. Niech \(\displaystyle{ r>1}\) będzie liczbą naturalną. Udowodnić, iż ciąg reszt modulo \(\displaystyle{ r}\) kolejnych wyrazów ciągu Fibonacciego jest okresowy
m-z
46. Niech \(\displaystyle{ NWD(a, b)=d}\) oraz \(\displaystyle{ NWD(a^{\prime}, b^{\prime})=d^{\prime}}\). Wykazać, że \(\displaystyle{ NWD(a a^{\prime}, ab^{\prime}, b a^{\prime}, b b^{\prime})=d d^{\prime}}\)
mz
47. Wykazać, iż istnieje nieskończenie wiele liczb pierwszych równych \(\displaystyle{ a^2+b^2+c^2 +1}\) gdzie \(\displaystyle{ a, b, c}\) są liczbami całkowitymi
m-z
48. Niech \(\displaystyle{ a, b, c}\) będą liczbami całkowitymi takimi, że \(\displaystyle{ a^{b+c} = cb^c}\) Wykazać, że \(\displaystyle{ b}\) dzieli \(\displaystyle{ c}\), oraz że \(\displaystyle{ c=d^b}\) gdzie \(\displaystyle{ d}\) jest liczba naturalną.
nz
49. Wyznaczyć wszystkie takie całkowite \(\displaystyle{ a, b}\) że \(\displaystyle{ a^2 + b^2}\) dzieli \(\displaystyle{ (a + b)(ab - 1)}\)

50. Wyznaczyć wszystkie pary \(\displaystyle{ (a, b)}\) ; \(\displaystyle{ a \neq b}\) liczb całkowitych, takich że
\(\displaystyle{ \begin{cases} a^2+ b= p^n\\a +b^2=p^m\end{cases}}\)
gdzie \(\displaystyle{ m}\) , \(\displaystyle{ n}\) są całkowite zaś \(\displaystyle{ p}\) jest liczbą pierwszą
51. Udowodnić, że jeśli liczby \(\displaystyle{ a, b}\) oraz \(\displaystyle{ \frac{a+1}{b} + \frac{b+1}{a}}\) są naturalne to \(\displaystyle{ NWD(a, b) \leq \sqrt{a+b}}\).
Czy może być równość (w tej nierówności) gdy \(\displaystyle{ NWD(a, b) < a < b}\) ?
52. Wykazać dla dowolnej liczby całkowitej nieujemnej \(\displaystyle{ k}\) istnieje liczba naturalna \(\displaystyle{ n_k}\) taka, że: \(\displaystyle{ (\sqrt{3} - \sqrt{2})^k = \sqrt{n_k} - \sqrt{n_k - 1}}\)
53. Czy istnieje liczba naturalna \(\displaystyle{ n}\) będąca kwadratem liczby całkowitej oraz taka, że suma cyfr liczby \(\displaystyle{ n}\) to \(\displaystyle{ 2013}\) ?

54. Wyznaczyć wszystkie trójki \(\displaystyle{ (a, b, c)}\) liczb naturalnych takie iż: \(\displaystyle{ a^3 + b^3 =22c^3}\)

55. Wykazać, że jeśli \(\displaystyle{ F_n \in P}\) to \(\displaystyle{ n \in \ P \backslash \{ 2 \}}\) lub \(\displaystyle{ n=4}\). Czy zachodzi też twierdzenie odwrotne ?
gdzie \(\displaystyle{ P}\) to zbiór liczb pierwszych, zaś \(\displaystyle{ F_n}\) jest ciągiem Fibonacciego
(m-z)
56. Udowodnić, że dla dowolnej liczby naturalnej \(\displaystyle{ k}\) istnieje liczba naturalna \(\displaystyle{ n}\) taka, iż w zbiorze \(\displaystyle{ (n, 2n)}\) jest co najmniej \(\displaystyle{ k}\) liczb pierwszych.
57. Wykazać, że dla każdej liczby naturalnej \(\displaystyle{ n > 1}\) istnieje liczba pierwsza, która dzieli dokładnie jedną liczbę naturalną nie większą od \(\displaystyle{ n}\)

58. Wykazać, że jeżeli \(\displaystyle{ a, b}\) są liczbami naturalnymi takimi, że liczba \(\displaystyle{ ab + 2}\) dzieli liczbę \(\displaystyle{ a ^2 + b ^2}\), to \(\displaystyle{ \frac{2(a^2 + b^2)}{ab + 2}}\) jest kwadratem liczby naturalnej.

59. Niech \(\displaystyle{ p}\) będzie liczbą pierwszą; Udowodnić, że równanie \(\displaystyle{ x^{p-1}+ y^{p-1} =z^p}\) ma rozwiązanie w liczbach naturalnych; oprócz trywialnego \(\displaystyle{ x=y=z=2}\)
60. Erdós problem; Wyznaczyć najmniejszą liczbę naturalną \(\displaystyle{ n}\) taką, iż \(\displaystyle{ 2^n - 7}\) jest pierwsza. Wykazać też, że takie \(\displaystyle{ n}\) nie może być równe \(\displaystyle{ 10k +7}\) ani \(\displaystyle{ 12k +11}\)
61. Wykazać, że hipoteza istnienia nieskończenie wielu par liczb pierwszych \(\displaystyle{ (p, p+2)}\) jest równoważna takiej hipotezie: równanie \(\displaystyle{ p+q = r}\) ma nieskończenie wiele rozwiązań w liczbach pierwszych \(\displaystyle{ p, q, r}\)
m-z
62. a) Rozstrzygnąć czy istnieją trzy różne liczby pierwsze \(\displaystyle{ q_j}\) dla \(\displaystyle{ j=1, 2, 3}\) iż: \(\displaystyle{ q_1^3 + q_2^3 + q_3^3 = q_1q_2q_3}\) ?
b) jw. gdy \(\displaystyle{ q_1^2 + q_2^2 + q_3^2 = q_1q_2q_3}\)
63. Czy dla każdej liczby naturalnej \(\displaystyle{ k}\) istnieje liczba całkowita \(\displaystyle{ n}\), taka, że \(\displaystyle{ \frac{3^n - 1}{10^k} \in Z}\);
gdzie \(\displaystyle{ Z= \{ 0, \pm 1, \pm2, ... \}}\)
64. Hipoteza Bombeliego: Nie istnieją liczby naturalne \(\displaystyle{ x, y, z, n}\) oraz \(\displaystyle{ n>2}\): \(\displaystyle{ {x \choose n}+ {y \choose n} = {z \choose n}}\)
Wykazać lub obalić te hipotezę.

65. Wskazać przykład liczb naturalnych \(\displaystyle{ m_1, …, m_n}\) (\(\displaystyle{ n \geq 3}\)) takich, że:
i) dla \(\displaystyle{ NWD(m_{i_1}, … , m_{i_k}) > 1}\) dla \(\displaystyle{ 1 < k < n}\) oraz \(\displaystyle{ 1 \leq i_1 < … < i_k \leq n}\)
ii) dla \(\displaystyle{ NWD(m_1, …, m_n) = 1}\)
66. a) Znaleźć pięć najmniejszych liczb naturalnych \(\displaystyle{ n}\) takich, że \(\displaystyle{ n^2 -1}\) jest iloczynem trzech różnych liczb pierwszych
b) Znaleźć taką liczbę \(\displaystyle{ n}\)\(\displaystyle{ n^2 + 1}\) jest iloczynem trzech różnych liczb pierwszych nieparzystych
67. Wykazać, że jeśli \(\displaystyle{ n^c}\) jest liczbą całkowitą dla \(\displaystyle{ n = 1, 2, ...}\) to \(\displaystyle{ c \in N \cup \{ 0 \}}\)
m-z
68. Udowodnić, iż gdy \(\displaystyle{ n}\) jest liczbą naturalną to \(\displaystyle{ \lfloor \frac{(n - 1)!}{n(n+1)} \rfloor}\) jest parzysta
69. Wykazać, iż dla każdej liczby naturalnej \(\displaystyle{ n>1}\) istnieje \(\displaystyle{ n}\) elementowy zbiór \(\displaystyle{ S}\) liczb całkowitych o tej własności, iż \(\displaystyle{ (a - b)^2}\) dzieli \(\displaystyle{ ab}\) dla \(\displaystyle{ a, b \in S}\) i \(\displaystyle{ a \neq b}\)
104 ntp
70. Niech \(\displaystyle{ a_i \neq a_j}\) dla \(\displaystyle{ i \neq j}\) będą liczbami całkowitymi. Wykazać, że iloczyn ułamków \(\displaystyle{ \frac{a_k - a_l}{k - l}}\) gdzie \(\displaystyle{ n \geq k > l \geq 1}\) jest liczbą całkowitą

71. a) Udowodnić, że gdy \(\displaystyle{ p}\) jest liczbą pierwszą, to \(\displaystyle{ {2p \choose p} \equiv 2 \ (mod \ p)}\)
b) Wykazać, że gdy ponadto \(\displaystyle{ p \geq 5}\) to \(\displaystyle{ (p! , (p-2)! -1) = p}\)
gdzie \(\displaystyle{ (a, b)}\) oznacza największy wspólny dzielnik \(\displaystyle{ a}\) i \(\displaystyle{ b}\)
72. Udowodnić ze liczba \(\displaystyle{ n> 1}\) jest pierwsza gdy ma jedyny rozkład: \(\displaystyle{ n = z - \frac{z}{xy+ x - y}}\) gdzie \(\displaystyle{ x, y, z}\) są liczbami naturalnymi.
np. gdy \(\displaystyle{ n=6}\) to np. \(\displaystyle{ (x, y, z)=(2, 5, 7)}\) lub \(\displaystyle{ (x, y, z)=(3, 2, 7)}\)

73. 7 x podzielność; Wykazać następujące kongruencje:
i) \(\displaystyle{ 20^{15} \equiv 1 \ (mod \ 11*31*61)}\)
ii) \(\displaystyle{ 2^{2^{38}} \equiv -3 \ (mod \ 19)}\)
iii) \(\displaystyle{ 5^{6614} - 12^{857} \equiv 1 \ (mod \ 7)}\)
iv) \(\displaystyle{ 2^{4619} \equiv 1 \ (mod \ 9239)}\)
v) \(\displaystyle{ 19^{8} \equiv 1 \ (mod \ 240)}\)
vi) \(\displaystyle{ 2^{64} \equiv 1 \ (mod \ 641)}\)
vii) czy istnieje liczba całkowita \(\displaystyle{ x}\) taka, że \(\displaystyle{ x^2 \equiv 52 \ (mod \ 159)}\) ?
74. Wykazać, że dla każdej liczby naturalnej \(\displaystyle{ k}\) istnieją dwie kolejne liczby naturalne, z których każda ma co najmniej \(\displaystyle{ k}\) różnych dzielników pierwszych
75. Wyznaczyć najmniejszą liczbę całkowitą nieujemną, która na trzy różne sposoby jest sumą kwadratów dwóch liczb całkowitych
76. Wykazać, że dla dowolnej liczby całkowite \(\displaystyle{ a>2}\) istnieje liczba pierwsza \(\displaystyle{ p}\) taka, że \(\displaystyle{ p}\) dzieli \(\displaystyle{ a^3 +1}\) ale \(\displaystyle{ p}\) nie dzieli \(\displaystyle{ a+1}\)
77. 7 x Diofantos; Rozwiąż równania (wszystkie zmienne są to liczby całkowite)
i) \(\displaystyle{ x^3 + 5 =117y^3}\)
ii) \(\displaystyle{ a^{b^2}= b^2}\), \(\displaystyle{ a > 1 \ b > 1}\)
iii) \(\displaystyle{ x^2+y^2= (x - y)^3}\)
iv) \(\displaystyle{ x^{2} + y^2 +5 = 5xy}\)
v) \(\displaystyle{ x^{3} + y^4 = 7}\)
vi) \(\displaystyle{ \lfloor x \rfloor + \lfloor p - x \rfloor = p}\), \(\displaystyle{ p}\) - liczba pierwsza
vii) \(\displaystyle{ x^{2} + 5 = y^3}\),
78. Wykazać, że istnieje nieskończenie wiele czwórek \(\displaystyle{ x, y, z, t}\) nie mających wspólnego dzielnika większego od \(\displaystyle{ 1}\) i takich, że \(\displaystyle{ x^3+y^3+z^2=t^4}\)
79. Jeśli \(\displaystyle{ m=\prod_{j} p_j^{\alpha_j}}\): rozkład na czynniki pierwsze, to niech \(\displaystyle{ f(m)=1 + \sum_j p_j\alpha_j}\)
Jakie liczby mogą się w ten sposób tj. przez iteracje \(\displaystyle{ m, f(m), f(f(m)), …}\); wygenerować nieskończoną ilość razy ?
80. Niech \(\displaystyle{ a, b >1}\) oraz \(\displaystyle{ D_{a^n - 1} \subset D_{b^n - 1}}\) dla dowolnego \(\displaystyle{ n > 1}\). Udowodnić iż \(\displaystyle{ b = a^j}\) dla jakiegoś \(\displaystyle{ j>1}\)
gdzie \(\displaystyle{ D_{n}}\) to zbiór dzielników liczby \(\displaystyle{ n}\)
81. Wykazać, iż istnieje liczba całkowita\(\displaystyle{ x}\) taka że \(\displaystyle{ x \equiv a \ (mod \ m)}\) oraz \(\displaystyle{ x \equiv b \ (mod \ n)}\) tylko wtedy, gdy \(\displaystyle{ a \equiv b \ (mod \ (m,n))}\)
m-z
82. Znaleźć największa liczbę naturalna, której nie można przedstawić jako \(\displaystyle{ 2x+ 3y +5z}\)
wn
83. Niech \(\displaystyle{ p \equiv 5 \ (mod \ 24)}\) będzie liczbą pierwszą. Wykazać, że jeśli \(\displaystyle{ p}\) nie dzieli \(\displaystyle{ a}\) to nie istnieje liczba całkowita \(\displaystyle{ x}\) taka, że \(\displaystyle{ x^2 \equiv 3a^2 (mod \ p)}\)
wn
84. Niech \(\displaystyle{ a}\) i \(\displaystyle{ b}\) będą liczbami całkowitymi takimi, że dla każdej liczby pierwszej \(\displaystyle{ p}\) istnieje liczba całkowita nieujemna \(\displaystyle{ x}\) taka, że \(\displaystyle{ p}\) dzieli \(\displaystyle{ a - b^x}\). Wykazać, że \(\displaystyle{ b=a^j}\) dla jakiejś liczby całkowitej \(\displaystyle{ j}\)

85. Wyznaczyć wszystkie pary \(\displaystyle{ (a, b)}\) liczb naturalnych takie, że \(\displaystyle{ ab}\) dzieli \(\displaystyle{ a^2 + b^2 + 3}\)

86. Wyznaczyć najmniejszą liczbę naturalną \(\displaystyle{ n}\) taką że \(\displaystyle{ 19n+1}\) oraz \(\displaystyle{ 95n+1}\) są kwadratami liczb całkowitych

87. Niech \(\displaystyle{ n>2}\) będzie liczbą naturalną; Wykazać, iż wśród wyrażeń: \(\displaystyle{ \frac{1}{n}, …. , \frac{n - 1}{n}}\)
jest parzysta ilość nieskracalnych
88. Wykazać iż \(\displaystyle{ 2^{2700} \equiv 1 (mod \ 2701)}\) (E. Lucas), mimo iż \(\displaystyle{ 2701=37*73 \notin P}\) tj. małego twierdzenie Fermata nie można odwrócić; i że więcej takich kontrprzykładów jest gdy \(\displaystyle{ n= pq}\) gdzie \(\displaystyle{ p}\) i \(\displaystyle{ q}\) są liczbami pierwszymi oraz \(\displaystyle{ p \equiv 1 (mod \ 4)}\) i \(\displaystyle{ q = 2p -1}\)
89. Czy istnieje nieskończenie wiele liczb naturalnych \(\displaystyle{ m}\) takich, iż \(\displaystyle{ 4^{27} + 4^{100} + 4^m}\) jest kwadratem liczby całkowitej ?
Jeśli nie, to wyznaczyć największą liczbę o takiej własności
90. Wykazać, że jeśli suma dzielników liczby całkowitej \(\displaystyle{ N>1}\) jest równa \(\displaystyle{ 2N+1}\) to \(\displaystyle{ N}\) jest kwadratem liczby nieparzystej.
91. Niech \(\displaystyle{ p, q, r}\) będą różnymi liczbami pierwszymi oraz
\(\displaystyle{ A = \{ p^aq^br^c : 0 \leq a, b, c \leq 5 \}}\)
Wyznaczyć najmniejsze \(\displaystyle{ n}\) takie, iż każdy \(\displaystyle{ n}\) elementowy podzbiór \(\displaystyle{ A}\) zawiera elementy \(\displaystyle{ x}\) i \(\displaystyle{ y}\); \(\displaystyle{ x \neq y}\) oraz \(\displaystyle{ x}\) dzieli \(\displaystyle{ y}\) bądź też \(\displaystyle{ y}\) dzieli \(\displaystyle{ x}\)
92. Niech \(\displaystyle{ a, b, n, m}\) będą liczbami całkowitymi dodatnimi, oraz \(\displaystyle{ n > 1}\). Udowodnić, że jeśli \(\displaystyle{ a^n + b^n = 2^m}\) to \(\displaystyle{ a = b}\)
musztari
93. Udowodnić, że istnieje nieskończenie wiele kolejnych liczb naturalnych (np. takich jak \(\displaystyle{ (8, 9)}\) czy \(\displaystyle{ (288, 289)}\)) o tej własności, iż dzielniki pierwsze każdej z tych liczb są co najmniej w drugiej potędze
94. Istnieje formuła Lagrange’a, która wyraża wykładnik z którym liczba pierwsza \(\displaystyle{ p}\) jest w rozkładzie na czynniki pierwsze \(\displaystyle{ n!}\)
Czy istnieje analog dla „multifaktoriala”:
\(\displaystyle{ n!…!_{k} = \prod_{j} n- kj}\)
w szczególności: iloma zerami kończy się liczba \(\displaystyle{ 2013!!!}\)
95. Wyznaczyć wszystkie liczby naturalne \(\displaystyle{ m}\) takie, że: \(\displaystyle{ d(m)^4 = m}\); gdzie \(\displaystyle{ d(m)}\) to ilość dzielników jakie ma \(\displaystyle{ m}\)
(liczbą taką jest np. \(\displaystyle{ m=3^8}\))
96. Niech \(\displaystyle{ a= 23! \sum_{j=1}^{23} \frac{1}{j}}\). Wyznaczyć resztę z dzielenia \(\displaystyle{ a}\) przez \(\displaystyle{ 13}\)
104 ntp
97. Znaleźć wszystkie takie \(\displaystyle{ m, \ n}\) liczby naturalne, iż \(\displaystyle{ \phi(m)}\) dzieli \(\displaystyle{ n}\) zaś \(\displaystyle{ \phi(n)}\) dzieli \(\displaystyle{ m}\)
a \(\displaystyle{ \phi()}\) to funkcja Eulera
98. Wykazać, że układ:
\(\displaystyle{ \begin{cases} x^2+ 6y^2 = z^2\\6x^2 + y^2 = w^2\end{cases}}\)
nie ma rozwiązań w \(\displaystyle{ N}\)
99. Niech \(\displaystyle{ A_1, A_2, A_3}\) będą zbiorami niepustymi liczb całkowitych takimi, że dla \(\displaystyle{ \{ i, j, k \} = \{ 1, 2, 3 \}}\):
\(\displaystyle{ (x \in A_i, y \in A_j) \Rightarrow (x+y \in A_k, x-y \in A_k )}\)
Udowodnić, że choć dwa ze zbiorów \(\displaystyle{ A_1, A_2, A_3}\) są równe. Czy mogą być niektóre z tych zbiorów rozłączne ?
100. Niech \(\displaystyle{ f: N \mapsto N}\) będzie funkcją taką że \(\displaystyle{ af(a) + bf(b) + 2ab}\) jest kwadratem (liczby całkowitej) dla dowolnych \(\displaystyle{ a, b \in N}\). Wykazać, iż \(\displaystyle{ f(m)=m}\) dla \(\displaystyle{ m \in N}\)
101. Udowodnić, iż jeśli \(\displaystyle{ a}\) i \(\displaystyle{ b}\) są liczbami naturalnymi, takimi iż \(\displaystyle{ 4ab - 1}\) dzieli \(\displaystyle{ (4a^2 - 1)^2}\) to \(\displaystyle{ a=b}\)
102. Hipoteza Lagrange’a: Każda liczba nieparzysta \(\displaystyle{ n>5}\) jest równa \(\displaystyle{ n= p+ 2q}\) gdzie \(\displaystyle{ p, q \in P}\) . Czy \(\displaystyle{ p}\) i \(\displaystyle{ q}\) istnieją gdy \(\displaystyle{ n = 2013}\) ?
103. Wykazać że \(\displaystyle{ p_{n+1} + p_{n+2} \leq p_1….p_n}\) dla \(\displaystyle{ n>2}\)
gdzie \(\displaystyle{ p_n}\) to \(\displaystyle{ n}\) ta liczba pierwsza
Ukryta treść:    
Awatar użytkownika
Vax
Użytkownik
Użytkownik
Posty: 2913
Rejestracja: 27 kwie 2010, o 22:07
Płeć: Mężczyzna
Lokalizacja: Biała Podlaska / Warszawa
Podziękował: 4 razy
Pomógł: 612 razy

[MIX] Zestaw mola (końcówka)

Post autor: Vax »

1:    
4:    
7:    
13:    
14:    
24:    
33:    
35:    
37:    
63:    
71:    
74:    
76:    
83:    
87:    
88:    
92:    
96:    
98:    
103:    
Awatar użytkownika
yorgin
Użytkownik
Użytkownik
Posty: 12762
Rejestracja: 14 paź 2006, o 12:09
Płeć: Mężczyzna
Lokalizacja: Kraków
Podziękował: 17 razy
Pomógł: 3440 razy

[MIX] Zestaw mola (końcówka)

Post autor: yorgin »

17:    
21:    
Awatar użytkownika
Ponewor
Moderator
Moderator
Posty: 2218
Rejestracja: 30 sty 2012, o 21:05
Płeć: Mężczyzna
Lokalizacja: Warszawa
Podziękował: 70 razy
Pomógł: 297 razy

[MIX] Zestaw mola (końcówka)

Post autor: Ponewor »

Komentarz do 2:    
4:    
Kontrprzykład do 5:    
7:    
14:    
41 b:    
coś nie tak z 42:    
46:    
problem poboczny do 60:    
61:    
73 ii:    
73 vii:    
76:    
77 vii:    
coś nie tak z 82:    
Istotne wnioski do 85:    
92:    
101:    
102:    
bakala12
Użytkownik
Użytkownik
Posty: 3044
Rejestracja: 25 mar 2010, o 15:34
Płeć: Mężczyzna
Lokalizacja: Gołąb
Podziękował: 24 razy
Pomógł: 513 razy

[MIX] Zestaw mola (końcówka)

Post autor: bakala12 »

9.:    
12.:    
27.:    
53.:    
55.:    
.-- 19 lip 2013, o 18:27 --
44.:    
diana7
Użytkownik
Użytkownik
Posty: 78
Rejestracja: 17 lip 2012, o 20:38
Płeć: Kobieta
Lokalizacja: Bielsko-Biała
Pomógł: 13 razy

[MIX] Zestaw mola (końcówka)

Post autor: diana7 »

18:    
19:    
22:    
23:    
25, kontrprzykład:    
51:    
61:    
89:    
Ostatnio zmieniony 22 lip 2013, o 19:15 przez diana7, łącznie zmieniany 1 raz.
Awatar użytkownika
yorgin
Użytkownik
Użytkownik
Posty: 12762
Rejestracja: 14 paź 2006, o 12:09
Płeć: Mężczyzna
Lokalizacja: Kraków
Podziękował: 17 razy
Pomógł: 3440 razy

[MIX] Zestaw mola (końcówka)

Post autor: yorgin »

Ponewor pisze:
coś nie tak z 82:    
A może jednak?:    
Komentarz do 79:    
6:    
ordyh
Użytkownik
Użytkownik
Posty: 255
Rejestracja: 6 paź 2009, o 18:04
Płeć: Mężczyzna
Pomógł: 66 razy

[MIX] Zestaw mola (końcówka)

Post autor: ordyh »

58.:    
65.:    
Awatar użytkownika
Vax
Użytkownik
Użytkownik
Posty: 2913
Rejestracja: 27 kwie 2010, o 22:07
Płeć: Mężczyzna
Lokalizacja: Biała Podlaska / Warszawa
Podziękował: 4 razy
Pomógł: 612 razy

[MIX] Zestaw mola (końcówka)

Post autor: Vax »

ciekawe 11:    
Awatar użytkownika
mol_ksiazkowy
Użytkownik
Użytkownik
Posty: 11415
Rejestracja: 9 maja 2006, o 12:35
Płeć: Mężczyzna
Lokalizacja: Kraków
Podziękował: 3155 razy
Pomógł: 748 razy

[MIX] Zestaw mola (końcówka)

Post autor: mol_ksiazkowy »

ad 40 Zadanie ze \(\displaystyle{ 101}\) nierozwiązanych
Ukryta treść:    
Awatar użytkownika
rochaj
Użytkownik
Użytkownik
Posty: 411
Rejestracja: 3 lip 2012, o 23:51
Płeć: Mężczyzna
Lokalizacja: komp
Podziękował: 128 razy
Pomógł: 2 razy

[MIX] Zestaw mola (końcówka)

Post autor: rochaj »

czy w 64 coś jeszcze wiadomo o x,y,z?
Awatar użytkownika
mol_ksiazkowy
Użytkownik
Użytkownik
Posty: 11415
Rejestracja: 9 maja 2006, o 12:35
Płeć: Mężczyzna
Lokalizacja: Kraków
Podziękował: 3155 razy
Pomógł: 748 razy

[MIX] Zestaw mola (końcówka)

Post autor: mol_ksiazkowy »

czy w 64 coś jeszcze wiadomo o x,y,z?
to mają być liczby naturalne, zas n też i \(\displaystyle{ n \geq 3}\)
Awatar użytkownika
rochaj
Użytkownik
Użytkownik
Posty: 411
Rejestracja: 3 lip 2012, o 23:51
Płeć: Mężczyzna
Lokalizacja: komp
Podziękował: 128 razy
Pomógł: 2 razy

[MIX] Zestaw mola (końcówka)

Post autor: rochaj »

mol_ksiazkowy pisze:
czy w 64 coś jeszcze wiadomo o x,y,z?
to mają być liczby naturalne, zas n też i \(\displaystyle{ n \geq 3}\)
Bardziej chodziło mi o to czy mogą być sobie równe, \(\displaystyle{ x=y=5,z=6,n=3}\).
ODPOWIEDZ